6
$\begingroup$

A theorem of Hopkins and Mahowald states that the Thom spectrum of the map $\Omega^2 S^3 \to B\mathrm{GL}_1(\mathbb{S}_{(p)})$ classifying the element $p$ is exactly $\mathrm{H}\mathbf{F}_p$. Let $T(1)$ denote the free $\mathbf{E}_1$-ring with $\alpha_1 = 0$ (so that at $p=2$, it is the 2-localization of the $\mathbf{E}_2$(!)-ring spectrum $X(2)$). The following question stemmed from attempting to understand whether there is a $p$-local orientation $T(1) \to \mathrm{H}\mathbf{F}_p$ which is a map of Thom spectra, i.e., if there is an orientation which comes from Thom-ifying a map of spaces $\Omega S^{2p-1} \to \Omega^2 S^3$ over $B\mathrm{GL}_1(\mathbb{S}_{(p)})$.

The element $\alpha_1\in \pi_{2p-3}(\mathbb{S})$ desuspends to an element $\alpha_1'$ of the unstable homotopy group $\pi_{2p}(S^3)$. The unstable element $\alpha_1'$ gets us a map $S^{2p-2} \to \Omega^2 S^3$, and hence a map $\Omega S^{2p-1} \to \Omega^2 S^3$. Does this Thom-ify to an orientation $T(1) \to \mathrm{H}\mathbf{F}_p$? This would follow, I think, if we knew that the map $\Omega^2 S^3 \to B\mathrm{GL}_1(\mathbb{S}_{(p)})$ is an isomorphism on $\pi_{2p-2}$, but I don't know if this is true.

A related question is the following. The map $\Omega S^{2p-1} \to \Omega^2 S^3$ is in turn is adjoint to a map $\Sigma \Omega S^{2p-1} \to \Omega S^3$. The James splitting tells us that the source splits as $\bigvee_{n\geq 0} S^{2n(p-1)+1}$, so we get maps $S^{2n(p-1)+1} \to \Omega S^3$, which are exactly elements of $\pi_{2n(p-1)+2}(S^3)$. Are these elements (nonzero multiples of) the desuspensions of the other $\alpha$-family elements $\alpha_n$?

$\endgroup$
5
  • $\begingroup$ I think there is a kind of answer in page 107-114 of Ravenel's orange book, or something which you can extract your answer from it. There is also Paul Turner's Manchester thesis which some work on T(n) spectra and mod $2$ orientations is done, but I am not sure where to find a pdf file! $\endgroup$
    – user51223
    Dec 2, 2018 at 5:48
  • $\begingroup$ And if I remember correctly, $T(i)$ spectra interpolate between $X(n)$ and $X(n+1)$, so aren't they just the $F(n)$ spectra defined in Revanel's Orange book? $\endgroup$
    – user51223
    Dec 2, 2018 at 5:54
  • $\begingroup$ @user51223 Thanks. I know of that part of Ravenel's book, but I don't know how to use it to answer my question. Where can I find a copy of his thesis? It seems really hard to find a copy online. The spectra T(i) do not interpolate between X(n) and X(n+1); instead, they split off of X(p^i)_p much in the same way as BP splits off of MU_p. $\endgroup$
    – skd
    Dec 2, 2018 at 15:22
  • $\begingroup$ @user51223 I think I have a copy of Paul Turner's thesis in my office, which I can check tomorrow, but I am fairly sure that it does not contain the material that you mention. Perhaps you are misremembering where you saw it? $\endgroup$ Dec 2, 2018 at 22:25
  • $\begingroup$ @ Neil Strickland Yes, it is quite possible that I have been misremembering this. At the time of writing the above comment, I thought that it is in his thesis where he talks about HZ/2 or MO orientations and formal groups laws related to it! Unfortunately, I did not make a copy of his thesis! $\endgroup$
    – user51223
    Dec 3, 2018 at 7:08

1 Answer 1

1
$\begingroup$

This is a long comment about the ``existence of orientation'' part of your question, not a precise answer!

It seems to me the answer is negative (although I was trying to prove it is positive!). To see this, let's note that in your question, $GL_1(\mathbb{S})$ is the same as $SG$ (sometimes also denoted $F$) which the space of stable maps $S^0\to S^0$ of degree $1$ and the monoid strcuture is the multiplicative one arising from composition of stable maps. As a space $SG$ is homotopy equivalent to $Q_0S^0$ where $QS^0=\mathrm{colimit}\ \Omega^iS^i$ and $Q_iS^0$ denotes the path component corresponding to $i\in\pi_0QS^0\simeq\pi_0^{st}\simeq\mathbb{Z}$.

Now, for $\alpha_1\in {_p\pi_{2p-3}^{st}}$ viewed as a mapping $$S^{2p-3}\to Q_0S^0\simeq Q_1S^0=SG=\Omega(BSG)$$ and its adjoint $S^{2p-2}\to BSG$ the infinite loop structure on $BSG$, allows to extend this to a loop map $\Omega S^{2p-1}\to BSG$ and it seems that you want $T(1)$ to be the Thom spectrum of this map, right?

Let's take the element $S^1\to BSG$ whose Thom spectrum is $H\mathbb{F}_p$. If my understanding is correct, then in order to show that an orientation as in your question exists, it would be enough to show that the composition $$S^{2p-2}\stackrel{\alpha'}{\to} \Omega S^3\to BSG\ (*)$$ is the same as $\alpha$.

It think there could be different ways to get a contradiction to the existence of such a decomposition. For instance, it shows that $\alpha_1$ is a decomposable element in the ring $\pi_*^{st}$ which I presume it is not!

Moreover, after using loop structure of $BSG$ to extend $(*)$ to a composition of loop maps as $$\Omega S^{2p-1}\stackrel{\alpha'}{\to} \Omega^2S^3\to BSG$$ then upon Thomifying one gets $T(1)\to H\mathbb{F}_p$ as desired. Since you know about the pull back of $\alpha$ to an element of $\pi_{2p}S^3$ then it would be enough to show that looping $\Omega^2S^3\to BSG$ one gets the inclusion $\Omega^3S^3\to SG=Q_1S^0$. Note that from the beginning, since $S^{2p-3}$ is path connected for $p>1$ then we could work with $\Omega^3_0S^3$. Now, I am not sure about the last claim that the loop maps is the inclusion! Perhaps, using multiplicative structure on $SG$ would give the desired contradiction!

These might lead to an answer, I guess!

ADDED The $\alpha_n$ elements live in ${_2\pi_*}J$ which coincide with the image of the $J$ homomorphism $SO\to SG$ is $p>2$. Since you are already using James-splitting, then one way to think about this is to consider the adjoint mapping $f:S^{2n(p-1)+1}\to\Omega S^3$ and compose it with some suitable James-Hopf map, $j_t:\Omega S^3\to \Omega S^{2t+1}$ and see if the composition is nontrivial or not. The image of $J$ is known, and if the composition $j_t\circ f$ is nontrivial then there might be a chance that this gives you the $\alpha_n$ elements. On the other hand, since somehow these elements arise from odd-primary Hopd invariant one problem then by analogy first look at the prime $p=2$ and see starting from $\eta$, $\nu$, or $\sigma$, you can get a family whose all elements are nontrivial, and if so then your guess might be correct.

$\endgroup$
8
  • $\begingroup$ Thanks. I believe that what you wrote actually hints that the answer is probably positive. Namely, looping the map $\Omega^2 S^3 \to B\mathrm{GL}_1(\mathbb{S})$ gives the map $\Omega^3 S^3 \to \mathrm{GL}_1(\mathbb{S}) \subseteq \text{colim }\Omega^n S^n$. This factors through the map $\Omega^3 S^3 \to \Omega^3 S^3$, which is multiplication by (1-p) on homotopy. In particular, taking $\pi_{2p-3}$ gives a map $\pi_{2p}(S^3) \to \pi_{2p-3}(\mathbb{S})$ which sends $\alpha_1'$ to $\Sigma^\infty((1-p)\alpha_1')$. This map is clearly not the inclusion, but it just multiplies by the unit (1-p). $\endgroup$
    – skd
    Dec 3, 2018 at 14:52
  • $\begingroup$ well, I thought it is a decomposability argument and hence gives a negative answer! $\endgroup$
    – user51223
    Dec 3, 2018 at 16:39
  • $\begingroup$ Sorry, I'm not sure I understand why $\alpha_1$ would be a decomposable element. $\endgroup$
    – skd
    Dec 3, 2018 at 16:43
  • $\begingroup$ You're right! At the beginning, I was taking a map $S^3\to BSG$ and factorise $\alpha$ through it. I now think that the above arguments prove what you were after. Bearing in mind that $S^1\to BSG$ corresponds to $1-p\in{_p\pi_{2p-3}^{st}}\simeq\mathbb{Z}/p$ so the above composition indeed would be equal to $\alpha_1$ as you say and the orientation follows! I was just not sure about the identification of $T(1)$ as a Thom spectrum as I have assumed you are using as you say it has $\alpha_1=0$ which I was not sure about its meaning! $\endgroup$
    – user51223
    Dec 4, 2018 at 5:42
  • $\begingroup$ Thanks again. Note that $T(1)$ is defined to be the free $\mathbf{E}_1$-ring with $\alpha_1 = 0$, so it is defined as the Thom spectrum of that map. I guess the second part of my question is still unanswered. Do you have any thoughts on that? $\endgroup$
    – skd
    Dec 4, 2018 at 5:47

Your Answer

By clicking “Post Your Answer”, you agree to our terms of service and acknowledge that you have read and understand our privacy policy and code of conduct.

Not the answer you're looking for? Browse other questions tagged or ask your own question.